LSAT and Law School Admissions Forum

Get expert LSAT preparation and law school admissions advice from PowerScore Test Preparation.

 Administrator
PowerScore Staff
  • PowerScore Staff
  • Posts: 8917
  • Joined: Feb 02, 2011
|
#38158
Complete Question Explanation
(The complete setup for this game can be found here: lsat/viewtopic.php?t=14979)

The correct answer choice is (B)

If neither of the last two books discussed is summarized, we can conclude that neither K nor R is summarized:
PT71_D13 LG Explanations_Game #3_#15_diagram 1.png
Our diagram helps eliminate answer choices (A) and (C).

Next, recall that if R is not summarized, then N must be summarized (contrapositive of the second rule). In this instance, R is not summarized, and so N must be summarized—eliminating answer choice (E). Answer choice (D) is also incorrect, because if N were summarized along with both F and T, we would invariably end up with three consecutive books that are all summarized, in violation of the first rule. By the process of elimination, we can determine that answer choice (B) is correct.
You do not have the required permissions to view the files attached to this post.

Get the most out of your LSAT Prep Plus subscription.

Analyze and track your performance with our Testing and Analytics Package.